Đến nội dung

Hình ảnh

Topic về Bất đẳng thức, cực trị THCS


  • Please log in to reply
Chủ đề này có 1205 trả lời

#1141
NHoang1608

NHoang1608

    Sĩ quan

  • Thành viên
  • 375 Bài viết

1) Đặt $(x,y,z)=(\frac{a}{b},\frac{b}{c},\frac{c}{a})$. Khi đó $xyz=1, (a+b+c)(\frac{1}{a}+\frac{1}{b}+\frac{1}{c})=x+y+z+xy+yz+zx+3$

Bất đẳng thức tương đương: $(x+y+z)^{2}\geq x+y+z+xy+yz+zx+3$

                                          $\Leftrightarrow x^{2}+y^{2}+z^{2}+xy+yz+zx\geq 3+(x+y+z)$  $(1)$

Mặt khác $xy+yz+zx\geq 3\sqrt[3]{x^{2}y^{2}z^{2}}=3$

              $x^{2}+y^{2}+z^{2}\geq \frac{(a+b+c)^{2}}{3}\geq (x+y+z)\sqrt[3]{xyz}=x+y+z$

Suy ra bất đẳng thức $(1)$ đúng hay ta có ĐPCM


The greatest danger for most of us is not that our aim is too high and we miss it, but that it is too low and we reach it.

----- Michelangelo----


#1142
viet9a14124869

viet9a14124869

    Trung úy

  • Thành viên
  • 903 Bài viết

1. Cho 3 số  dưeơng a,b,c. CMR: $(\frac{a}{b}+\frac{b}{c}+\frac{c}{a})^{2}\geq(a+b+c)(\frac{1}{a}+\frac{1}{b}+\frac{1}{c})$

2. Cho các só thực x,y, z thỏa mãn điều kiện: x2 +y+z2=1. Tìm GTLN của A=xy+yz+2xz

3. Cho các số thực a,b,c thuộc đoạn [-2;5] tm: a+2b+3c$\leq$2. Tìm GTLN: a2+2b2+3c2

4. Cho a,b,c>0 tm: $\frac{1}{a}+\frac{1}{b}+\frac{1}{c}$=1. CMR:$\frac{a^{2}}{a+bc}+\frac{b^{2}}{b+ca}+\frac{c^{2}}{c+ab}\geq \frac{a+b+c}{4}$

Bài 2 , Ta có $\left\{\begin{matrix} xy\leq \frac{\sqrt{3}-1}{2}x^2+\frac{1}{2\sqrt{3}-2}y^2 & & \\ yz\leq \frac{\sqrt{3}-1}{2}z^2+\frac{1}{2\sqrt{3}-2}y^2 & & \\ 2xz\leq x^2+z^2 & & \end{matrix}\right.\Rightarrow xy+yz+2zx\leq \frac{\sqrt{3}+1}{2}(x^2+y^2+z^2)=\frac{\sqrt{3}+1}{2}\Leftrightarrow x=z=\frac{\sqrt{3}+1}{2}y\Leftrightarrow .........$

Bài 3 , $-2\leq a\leq 5\Leftrightarrow a^2\leq 3a+10 \Rightarrow ..... \Rightarrow a^2+2b^2+3c^2\leq (3a+10)+2(3b+10)+3(3c+10)=3(a+2b+3c)+60\leq  66\Leftrightarrow a=-2,b=5,c=-2$

Bài 4 , Chắc là dùng $\frac{a^2}{a+bc}=a-\frac{abc}{a+bc}$ rồi dùng AM-GM ở mẫu ^_^


                                                                    SÓNG BẮT ĐẦU TỪ GIÓ

                                                                    GIÓ BẮT ĐẦU TỪ ĐÂU ?

                                                                    ANH CŨNG KHÔNG BIẾT NỮA 

                                                                    KHI NÀO...? TA YÊU NHAU .


#1143
NHoang1608

NHoang1608

    Sĩ quan

  • Thành viên
  • 375 Bài viết

Bài 4 , Chắc là dùng $\frac{a^2}{a+bc}=a-\frac{abc}{a+bc}$ rồi dùng AM-GM 

Nếu sử dụng $AM-GM$ dưới mẫu sẽ không thỏa mãn dấu bằng vì $a=b=c=3$ mà.


The greatest danger for most of us is not that our aim is too high and we miss it, but that it is too low and we reach it.

----- Michelangelo----


#1144
viet9a14124869

viet9a14124869

    Trung úy

  • Thành viên
  • 903 Bài viết

Nếu sử dụng $AM-GM$ dưới mẫu sẽ không thỏa mãn dấu bằng vì $a=b=c=3$ mà.

Bạn tách ra $a+bc=a+\frac{bc}{3}+\frac{bc}{3}+\frac{bc}{3}$ rồi dùng AM-GM  là OK


                                                                    SÓNG BẮT ĐẦU TỪ GIÓ

                                                                    GIÓ BẮT ĐẦU TỪ ĐÂU ?

                                                                    ANH CŨNG KHÔNG BIẾT NỮA 

                                                                    KHI NÀO...? TA YÊU NHAU .


#1145
TrBaoChis

TrBaoChis

    Hạ sĩ

  • Banned
  • 81 Bài viết

1. Cho 3 số dương a,b,c. CMR: $(\frac{a}{b}+\frac{b}{c}+\frac{c}{a})^{2}\geq(a+b+c)(\frac{1}{a}+\frac{1}{b}+\frac{1}{c})$

2. Cho các só thực x,y, z thỏa mãn điều kiện: x2 +y+z2=1. Tìm GTLN của A=xy+yz+2xz

3. Cho các số thực a,b,c thuộc đoạn [-2;5] tm: a+2b+3c$\leq$2. Tìm GTLN: a2+2b2+3c2

4. Cho a,b,c>0 tm: $\frac{1}{a}+\frac{1}{b}+\frac{1}{c}$=1. CMR:$\frac{a^{2}}{a+bc}+\frac{b^{2}}{b+ca}+\frac{c^{2}}{c+ab}\geq \frac{a+b+c}{4}$

bài 4 từ gt suy ra ab+bc+ca=abc
thay vào bài sigma a^2/(a+bc) = sigma a^3/(a^2+abc)= sig ma a^3/(a+b)(a+c) 
đến đây amgm 
a^3/(a+b)(a+c) + a+b/8 +b+c/8



#1146
AnhTran2911

AnhTran2911

    Thượng sĩ

  • Thành viên
  • 230 Bài viết

bài 4 từ gt suy ra ab+bc+ca=abc
thay vào bài sigma a^2/(a+bc) = sigma a^3/(a^2+abc)= sig ma a^3/(a+b)(a+c) 
đến đây amgm 
a^3/(a+b)(a+c) + a+b/8 +b+c/8

Để tránh spam và làm mất đẹp topic, rất mong bạn sử dụng latex trong mỗi bài viết của mình. Thân.


        AQ02

                                 


#1147
haccau

haccau

    Binh nhất

  • Thành viên
  • 47 Bài viết

bài 4 từ gt suy ra ab+bc+ca=abc
thay vào bài sigma a^2/(a+bc) = sigma a^3/(a^2+abc)= sig ma a^3/(a+b)(a+c) 
đến đây amgm 
a^3/(a+b)(a+c) + a+b/8 +b+c/8

Viết lại giúp mình dễ hiểu nha bạn. Cảm ơn nhiều


:lol:  :lol:  :lol: Don't let your dreams just be dreams!!! :lol:  :lol:  :lol: 


#1148
TrBaoChis

TrBaoChis

    Hạ sĩ

  • Banned
  • 81 Bài viết

Viết lại giúp mình dễ hiểu nha bạn. Cảm ơn nhiều

$THEO$ $Ý$ $BẠN$
$\sum$ $\frac{a^2}{a+bc}$ = $\sum$ $\frac{a^3}{a^2+abc}$ = $\sum$ $\frac{a^3}{ab+bc+ca+a^2}$ = $\sum$ $\frac{a^3}{(a+b)(b+c)}$
$theo$ $AM-GM$ : $\sum$ $\frac{a^3}{(a+b)(b+c)}$ + $\sum$ $\frac{a+b}{8}$ + $\sum $ $\frac{a+b}{8}$ $\geq $ $3$ $\sum$ $\frac{a}{4}$ 
$ \rightarrow$ $\sum $ $\frac{a^2}{a+bc}$ $\geq$ $\frac{a+b+c}{4}$  
đúng thì like

Bài viết đã được chỉnh sửa nội dung bởi TrBaoChis: 21-04-2017 - 17:25


#1149
AnhTran2911

AnhTran2911

    Thượng sĩ

  • Thành viên
  • 230 Bài viết

Nhờ mọi người làm giúp ạ:

Cho a,b,c dương thỏa mãn a+b+c=3

CMR : $\sum\frac{a}{b+c^2}\geq\frac{3}{2}$


        AQ02

                                 


#1150
canletgo

canletgo

    Sĩ quan

  • Thành viên
  • 389 Bài viết

Mọi người làm hộ mk bài này với...cảm ơn nhiều  :)

Cho a, b, c và a + b + c $\leq$ 1

CMR: $\sqrt{a^{2}+\frac{1}{a^{2}}}+\sqrt{b^{2}+\frac{1}{b^{2}}}+\sqrt{c^{2}+\frac{1}{c^{2}}}\geq \sqrt{82}$


Bài viết đã được chỉnh sửa nội dung bởi canletgo: 10-05-2017 - 21:10

Alpha $\alpha$ 


#1151
05479865132

05479865132

    Binh nhì

  • Thành viên mới
  • 19 Bài viết

Cho ba số thực dương a,b,c thỏa mãn (a+b)(a+c)=1.Chứng minh các bất đẳng thức sau:

a)abc(a+b+c)$\leq \frac{1}{4}$

b)a(ab+bc+ca)$\leq \frac{2\sqrt{3}}{9}$


Bài viết đã được chỉnh sửa nội dung bởi 05479865132: 11-05-2017 - 23:11


#1152
maimaipham

maimaipham

    Lính mới

  • Thành viên mới
  • 1 Bài viết

mình học kém toán lắm bạn nào giải giúp mình nhé, chi tiết để mình còn hiểu:

a) với a, b là các số dương. cmr: a+b trên ab > hoặc = 4 trên a+b

b) cho các số thực dương x,y,z thỏa mãn x+y+z=4. cmr 1 trên xy + 1 trên xz > hoặc = 1



#1153
canletgo

canletgo

    Sĩ quan

  • Thành viên
  • 389 Bài viết

help meeeeeeeeeeeeeeeeeeeeeeee !!!

tìm min: $y=\frac{2}{1-x}+\frac{1}{x}$ 

với 0 < x < 1


Alpha $\alpha$ 


#1154
Cuongpa

Cuongpa

    Thượng sĩ

  • Thành viên
  • 238 Bài viết

help meeeeeeeeeeeeeeeeeeeeeeee !!!

tìm min: $y=\frac{2}{1-x}+\frac{1}{x}$ 

với 0 < x < 1

Ta có: 

$y=\frac{2}{1-x}+\frac{1}{x}=\frac{2x}{1-x}+\frac{1-x}{x}+3\geq 3+2\sqrt{2}$

Dấu $=$ xảy ra khi $\left\{\begin{matrix} 2x^2=(1-x)^2\\ 0<x<1 \end{matrix}\right.\Leftrightarrow x=\sqrt{2}-1$

 


Success doesn't come to you. You come to it.


#1155
manhhung2013

manhhung2013

    Sĩ quan

  • Thành viên
  • 306 Bài viết

Ta có: 

$y=\frac{2}{1-x}+\frac{1}{x}=\frac{2x}{1-x}+\frac{1-x}{x}+3\geq 3+2\sqrt{2}$

Dấu $=$ xảy ra khi $\left\{\begin{matrix} 2x^2=(1-x)^2\\ 0<x<1 \end{matrix}\right.\Leftrightarrow x=\sqrt{2}-1$

 

Ờ m được lắm.

 

Đóng góp cho topic một bài:

CMR với mọi a,b ,c>0:

$\frac{a}{\sqrt[3]{a^{3}+b^{3}}}+\frac{b}{\sqrt[3]{b^{3}+c^{3}}}+\frac{c}{\sqrt[3]{c^{3}+a^{3}}}\leqslant \frac{3}{\sqrt[3]{2}}$


đừng nghĩ LIKE và LOVE giống nhau...
giữa LIKE và LOVE chữ cái I đã chuyển thành O,tức là Important:quan trọng đã trở thành Only:duy nhất.
chữ cái K đã chuyển thành V:Keen:say mê đã trở thành Vascurla :ăn vào mạch máu.
vì thế đừng hỏi tại sao
lim(LIKE)=LOVE nhưng lim(LOVE) =

 


#1156
05479865132

05479865132

    Binh nhì

  • Thành viên mới
  • 19 Bài viết

Cho các số dương x,y,z thỏa mãn x+y+z$\leq$3.Tìm giá trị lớn nhất của biểu thức:

A=$\sqrt{1+x^2}+\sqrt{1+y^2}+\sqrt{1+z^2}+2(\sqrt{x}+\sqrt{y}+\sqrt{z})$

Cho số thực dương x,y,z thỏa mãn điều kiện x+y+z=1.Tìm giá trị nhỏ nhất của biểu thức:

$A=\frac{x^4}{(x^2+y^2)(x+y)}+\frac{y^4}{(y^2+z^2)(y+z)}+\frac{z^4}{(z^2+x^2)(z+x)}$



#1157
man2008

man2008

    Lính mới

  • Thành viên
  • 3 Bài viết

Các bạn giúp mình với :

Cho a,b,c dương. chứng minh :

a.$\frac{a^{5}}{b^{3}}+\frac{b^{5}}{c^{3}}+\frac{c^{5}}{a^{3}}\geq \frac{a^{3}}{b}+\frac{b^{3}}{c}+\frac{c^{3}}{a}$

 

b. $\frac{a^{5}}{b^{2}}+\frac{b^{5}}{c^{2}}+\frac{c^{5}}{a^{2}}\leq a^{2}b+b^{2}c+c^{2}a$

 

Chỉ sử dung phép biến đổi tương đương và bất đẳng thức phụ $a^{m+n}+b^{m+n}\geq a^{m}b^{n}+ a^{n}b^{m}$


Bài viết đã được chỉnh sửa nội dung bởi man2008: 19-05-2017 - 17:05


#1158
manhhung2013

manhhung2013

    Sĩ quan

  • Thành viên
  • 306 Bài viết

Cho các số dương x,y,z thỏa mãn x+y+z$\leq$3.Tìm giá trị lớn nhất của biểu thức:

A=$\sqrt{1+x^2}+\sqrt{1+y^2}+\sqrt{1+z^2}+2(\sqrt{x}+\sqrt{y}+\sqrt{z})$

 

 

Áp dụng BĐT:$\sqrt{x}+\sqrt{y}\leq \sqrt{2(x+y)}\Rightarrow \sum \left( \sqrt{x^{2}+1}+\sqrt{2x} \right )+(2-\sqrt{2})(\sqrt{x}+\sqrt{y}+\sqrt{z})\leqslant \sqrt{2}((x+1)+(y+1)+(z+1))+(2-\sqrt{2})(\frac{x+1}{2}+\frac{y+1}{2}+\frac{z+1}{2})\leqslant 6\sqrt{2}+3(2-\sqrt{2})=6+3\sqrt{2}$

Dấu = khi x=y=z=1


đừng nghĩ LIKE và LOVE giống nhau...
giữa LIKE và LOVE chữ cái I đã chuyển thành O,tức là Important:quan trọng đã trở thành Only:duy nhất.
chữ cái K đã chuyển thành V:Keen:say mê đã trở thành Vascurla :ăn vào mạch máu.
vì thế đừng hỏi tại sao
lim(LIKE)=LOVE nhưng lim(LOVE) =

 


#1159
haccau

haccau

    Binh nhất

  • Thành viên
  • 47 Bài viết

1. Cho a, b, c dương thỏa: 6a+3b+2c=abc. Tìm GTLN: $B=\frac{1}{\sqrt{a^{2}+1}}+\frac{2}{\sqrt{b^{2}+4}}+\frac{3}{\sqrt{c^{2}+9}}$

2. Cho các số a,b,c không âm. CMR: $a^{2}+b^{2}+c^{2}+3\sqrt[3]{(abc)^{2}} \geq 2(ab+bc+ca)$


:lol:  :lol:  :lol: Don't let your dreams just be dreams!!! :lol:  :lol:  :lol: 


#1160
AnhTran2911

AnhTran2911

    Thượng sĩ

  • Thành viên
  • 230 Bài viết

1. Cho a, b, c dương thỏa: 6a+3b+2c=abc. Tìm GTLN: $B=\frac{1}{\sqrt{a^{2}+1}}+\frac{2}{\sqrt{b^{2}+4}}+\frac{3}{\sqrt{c^{2}+9}}$

2. Cho các số a,b,c không âm. CMR: $a^{2}+b^{2}+c^{2}+3\sqrt[3]{(abc)^{2}} \geq 2(ab+bc+ca)$

Cái đầu chuyển từ $( a,b,c)\rightarrow(\frac{1}{a}; \frac{2}{b};\frac{3}{c})$ Đc bài toán quen thuộc

Cái thứ hai dùng Schur dạng pthức $\sum{a^2}+\frac{9abc}{a+b+c}\geq2\sum{ab}$


        AQ02

                                 





2 người đang xem chủ đề

0 thành viên, 2 khách, 0 thành viên ẩn danh